Difference between revisions of "2016 AMC 10A Problems/Problem 4"

m (Added 12A MAA box for matching 12A problem)
(8 intermediate revisions by 5 users not shown)
Line 5: Line 5:
 
<math>\textbf{(A) } -\frac{3}{8} \qquad \textbf{(B) } -\frac{1}{40} \qquad \textbf{(C) } 0 \qquad \textbf{(D) } \frac{3}{8} \qquad \textbf{(E) } \frac{31}{40}</math>
 
<math>\textbf{(A) } -\frac{3}{8} \qquad \textbf{(B) } -\frac{1}{40} \qquad \textbf{(C) } 0 \qquad \textbf{(D) } \frac{3}{8} \qquad \textbf{(E) } \frac{31}{40}</math>
  
==Solution==
+
==Solution 1==
  
 
The value, by definition, is <cmath>\begin{align*}
 
The value, by definition, is <cmath>\begin{align*}
Line 14: Line 14:
 
&= \frac{3}{8}-\left(-\frac{2}{5}\right)\left(-1\right)\\
 
&= \frac{3}{8}-\left(-\frac{2}{5}\right)\left(-1\right)\\
 
&= \frac{3}{8}-\frac{2}{5}\\
 
&= \frac{3}{8}-\frac{2}{5}\\
&= \boxed{\textbf{(B) } -\frac{1}{40}}
+
&= \boxed{\textbf{(B) } -\frac{1}{40}}.
 
\end{align*}</cmath>
 
\end{align*}</cmath>
 +
 +
==Solution 2==
 +
Do note that the denominator of the answer will be a multiple of 5 and 8 (40) and that the answer will also be negative. The only answer choice that satisfies this is <math>\boxed{B}</math>
 +
 +
==Video Solution==
 +
https://youtu.be/VIt6LnkV4_w?t=195
 +
 +
~IceMatrix
 +
 +
https://youtu.be/CrW0Yx5tqV0
 +
 +
~savannahsolver
  
 
==See Also==
 
==See Also==

Revision as of 18:17, 15 February 2021

Problem

The remainder can be defined for all real numbers $x$ and $y$ with $y \neq 0$ by \[\text{rem} (x ,y)=x-y\left \lfloor \frac{x}{y} \right \rfloor\]where $\left \lfloor \tfrac{x}{y} \right \rfloor$ denotes the greatest integer less than or equal to $\tfrac{x}{y}$. What is the value of $\text{rem} (\tfrac{3}{8}, -\tfrac{2}{5} )$?

$\textbf{(A) } -\frac{3}{8} \qquad \textbf{(B) } -\frac{1}{40} \qquad \textbf{(C) } 0 \qquad \textbf{(D) } \frac{3}{8} \qquad \textbf{(E) } \frac{31}{40}$

Solution 1

The value, by definition, is \begin{align*} \text{rem}\left(\frac{3}{8},-\frac{2}{5}\right) &= \frac{3}{8}-\left(-\frac{2}{5}\right)\left\lfloor\frac{\frac{3}{8}}{-\frac{2}{5}}\right\rfloor \\ &= \frac{3}{8}-\left(-\frac{2}{5}\right)\left\lfloor\frac{3}{8}\times\frac{-5}{2}\right\rfloor \\ &= \frac{3}{8}-\left(-\frac{2}{5}\right)\left\lfloor\frac{-15}{16}\right\rfloor\\ &= \frac{3}{8}-\left(-\frac{2}{5}\right)\left(-1\right)\\ &= \frac{3}{8}-\frac{2}{5}\\ &= \boxed{\textbf{(B) } -\frac{1}{40}}. \end{align*}

Solution 2

Do note that the denominator of the answer will be a multiple of 5 and 8 (40) and that the answer will also be negative. The only answer choice that satisfies this is $\boxed{B}$

Video Solution

https://youtu.be/VIt6LnkV4_w?t=195

~IceMatrix

https://youtu.be/CrW0Yx5tqV0

~savannahsolver

See Also

2016 AMC 10A (ProblemsAnswer KeyResources)
Preceded by
Problem 3
Followed by
Problem 5
1 2 3 4 5 6 7 8 9 10 11 12 13 14 15 16 17 18 19 20 21 22 23 24 25
All AMC 10 Problems and Solutions
2016 AMC 12A (ProblemsAnswer KeyResources)
Preceded by
Problem 2
Followed by
Problem 4
1 2 3 4 5 6 7 8 9 10 11 12 13 14 15 16 17 18 19 20 21 22 23 24 25
All AMC 12 Problems and Solutions

The problems on this page are copyrighted by the Mathematical Association of America's American Mathematics Competitions. AMC logo.png